Find the value of x in the triangle

Find The Value Of X In The Triangle

Answers

Answer 1

Answer:

x=62

Step-by-step explanation:

a triangle equals 180

so knowing that you subtract 56 from 180

That gives you 124

Then you divide it by 2 because the sides are equal.

That gives you 62

so x=62


Related Questions

Find the vertex of f(x)= x^2+ 6x + 36


Pls help soon

Answers

Answer:

vertex(-3,27)

Step-by-step explanation:

f(x)= x^2+ 6x + 36 ( a=1,b=6,c=36)

V(h,k)

h=-b/2a=-6/2=-3

k=f(-3)=3²+6(-3)+36

f(-3)=9-18+36=27

vertex(-3,27)

1. Suzette ran and biked for a total of 80 miles in 9 hours. Her average running speed was 5 miles per hour (mph) and her average biking speed was 12 mph. Let x = total hours Suzette ran. Let y = total hours Suzette biked. Use substitution to solve for x and y. Show your work. Check your solution. (a) How many hours did Suzette run? (b) How many hours did she bike?

Answers

Answer:

a) Suzette ran for 4 hours

b) Suzette biked for 5 hours

Step-by-step explanation:

Speed is rate of distance traveled, it is the ratio of distance traveled to time taken. It is given by:

Speed = distance / time

The total distance ran and biked by Suzette (d) = 80 miles, while the total time ran and biked by Suzette (t) = 9 hours.

For running:

Her speed was 5 miles per hour, let the total hours Suzette ran be x and the total distance she ran be p, hence since Speed = distance / time, therefore:

5 = p / x

p = 5x

For biking:

Her speed was 12 miles per hour, let the total hours Suzette ran be y and the total distance she ran be q, hence since Speed = distance / time, therefore:

12 = q / y

q = 12y

The total distance ran and biked by Suzette (d) = Distance biked + distance ran

d = p + q

80 = p + q

80 = 5x + 12y                 (1)

The total time taken to run and bike by Suzette (t) = time spent to bike + time spent to run

t = x + y

9 = x + y                         (2)

Solving equation 1 and equation 2, multiply equation 2 by 5 and subtract from equation 1:

7y = 35

y = 35/7

y = 5 hours

Put y = 5 in equation 2:

9 = x + 5

x = 9 -5

x = 4 hours

a) Suzette ran for 4 hours

b) Suzette biked for 5 hours

Consider the following system of equations: y=2x−2 6x+3y=2 The graph of these equations consists of two lines that: 1. intersect at more than one point. 2. intersect in an infinite number of points. 3. intersect at exactly one point. 4. do not intersect.

Answers

Answer:

3.  Intersect at exactly one point.  ( (2/3), (-2/3) )

Step-by-step explanation:

To make the comparison of these lines easier, let's rewrite the 2nd equation into slope-intercept form, as the 1st equation is in slope-intercept form.

[1] y = 2x - 2

---------------------

[2] 6x + 3y = 2 ==> 3y = 2 - 6x ==> y = -2x + (2/3)

[2] y = -2x + (2/3)

So now that we have both equations in slope-intercept form, we can see that the two equations are both linear, have different slopes, and have different y-intercepts.

Since these equations have both different slopes and different y-intercepts, we know that the lines will cross at least one point.  We can confirm that the lines only cross at a single point using the fact that both equations are linear, meaning there will only be one point of crossing.  To find that point, we can simply set the equations equal to each other.

y = 2x - 2

y = -2x + (2/3)

2x - 2 = -2x + (2/3)

4x = (8/3)

x = (8/12) = (2/3)

And plug this x value back into one of the equations:

y = 2x - 2

y = 2(2/3) - 2

y = (4/3) - (6/3)

y = (-2/3)

Thus these lines only cross at the point ( (2/3), (-2/3) ).

Cheers.

Answer:

I don't understand the question

Your mother has left you in charge of the annual family yard sale. Before she leaves you to your entrepreneurial abilities, she explains that she has made the job easy for you: everything costs either $1.50 or $3.50. She asks you to keep track of how many of each type of item is sold, and you make a list, but it gets lost sometime throughout the day. Just before she’s supposed to get home, you realize that all you know is that there were 150 items to start with (your mom counted) and you have 41 items left. Also, you know that you made $227.50. Write a system of equations that you could solve to figure out how many of each type of item you sold.

A) x + y = 109
(1.5)x + 227.50 = (3.5)y
B) x + y = 109
(3.5)x + 227.50 = (1.5)y
C) x + y = 41
(1.5)x + 227.50 = (3.5)y
D) x + y = 109
(1.5)x + (3.5)y = 227.50
E) x + y = 150
(1.5)x + (3.5)y = 227.50
F) x + y = $3.50
(1.5)x + (3.5)y = 227.50

Answers

Answer:

[tex]D)\ x + y = 109\\(1.5)x + (3.5)y = 227.50[/tex]

Step-by-step explanation:

Let the items sold with price $1.5 = [tex]x[/tex]

Let the items sold with price $3.5 = [tex]y[/tex]

Initially, total number of items = 150

Items left at the end of the day = 41

So, number of items sold throughout the day = Total number of items - Number of items left

Number of Items sold = 150 - 41 = 109

So, the first equation can be written as:

[tex]\bold{x+y = 109} ....... (1)[/tex]

Now, let us calculate the sales done by each item.

Sales from item with price $1.5 = Number of items sold [tex]\times[/tex] price of each item

= (1.5)[tex]x[/tex]

Sales from item with price $3.5 = Number of items sold [tex]\times[/tex] price of each item

= (3.5)[tex]y[/tex]

Total sales = [tex]\bold{(1.5)x+(3.5)y = 227.50} ....... (2)[/tex]

So, the correct answer is:

[tex]D)\ x + y = 109\\(1.5)x + (3.5)y = 227.50[/tex]

PLEaSE HELP!!!!!! will give brainliest to first answer

Answers

Answer:

The coordinates of A'C'S'T' are;

A'(-7, 2)

C'(-9, -1)

S'(-7, -4)

T'(-5, -1)

The correct option is;

B

Step-by-step explanation:

The coordinates of the given quadrilateral are;

A(-3, 1)

C(-5, -2)

S(-3, -5)

T(-1, -2)

The required transformation is T₍₋₄, ₁₎ which is equivalent to a movement of 4 units in the leftward direction and 1 unit upward

Therefore, we have;

A(-3, 1) + T₍₋₄, ₁₎ = A'(-7, 2)

C(-5, -2) + T₍₋₄, ₁₎ = C'(-9, -1)

S(-3, -5) + T₍₋₄, ₁₎ = S'(-7, -4)

T(-1, -2) + T₍₋₄, ₁₎ = T'(-5, -1)

Therefore, the correct option is B

PLEASE HELP
Find the area and the perimeter of the shaded regions below. Give your answer as a completely simplified exact value in terms of π (no approximations). The figures below are based on semicircles or quarter circles and problems b), c), and d) are involving portions of a square.

Answers

Answer:

perimeter is  4 sqrt(29) + 4pi  cm

area is 40 + 8pi cm^2

Step-by-step explanation:

We have a semicircle and a triangle

First the semicircle with diameter 8

A = 1/2 pi r^2 for a semicircle

r = d/2 = 8/2 =4

A = 1/2 pi ( 4)^2

  =1/2 pi *16

  = 8pi

Now the triangle with base 8 and height 10

A = 1/2 bh

  =1/2 8*10

  = 40

Add the areas together

A = 40 + 8pi cm^2

Now the perimeter

We have 1/2 of the circumference

1/2 C =1/2 pi *d

         = 1/2 pi 8

        = 4pi

Now we need to find the length of the hypotenuse of the right triangles

using the pythagorean theorem

a^2+b^2 = c^2

The base is 4 ( 1/2 of the diameter) and the height is 10

4^2 + 10 ^2 = c^2

16 + 100 = c^2

116 = c^2

sqrt(116) = c

2 sqrt(29) = c

Each hypotenuse is the same so we have

hypotenuse + hypotenuse + 1/2 circumference

2 sqrt(29) + 2 sqrt(29) + 4 pi

4 sqrt(29) + 4pi  cm

Step-by-step explanation:

First we need to deal with the half circle. The radius of this circle is 4, because the diameter is 8. The formula for the circumference of a circle is 2piR.

2pi4 so the perimeter for the half circle would be 8pi/2.

The area of that half circle would be piR^2 so 16pi/2.

Now moving on the triangle part, we need to find the hypotenuse side of AC. We will use the pythagoram theorem. 4^2+10^2=C^2

16+100=C^2

116=C^2

C=sqrt(116)

making the perimeter of this triangle 2×sqrt(116)

The area of this triangle is 8×10=80, than divided by 2 which is equal to 40.

We than just need to add up the perimeters and areas for both the half circle and triangle.

The area would be equal to 8pi+40

The perimeter would be equal to 4pi+4(sqrt(29))

Given the equations of a straight line f(x) (in slope-intercept form) and a parabola g(x) (in standard form), describe how to determine the number of intersection points, without finding the coordinates of such points. Do not give an example.

Answers

Answer:

Step-by-step explanation:

Hello, when you try to find the intersection point(s) you need to solve a system like this one

[tex]\begin{cases} y&= m * x + p }\\ y &= a*x^2 +b*x+c }\end{cases}[/tex]

So, you come up with a polynomial equation like.

[tex]ax^2+bx+c=mx+p\\\\ax^2+(b-m)x+c-p=0[/tex]

And then, we can estimate the discriminant.

[tex]\Delta=(b-m)^2-4*a*(c-p)[/tex]

If [tex]\Delta<0[/tex] there is no real solution, no intersection point.

If [tex]\Delta=0[/tex] there is one intersection point.

If [tex]\Delta>0[/tex] there are two real solutions, so two intersection points.

Hope this helps.

Complete the square to transform the expression x2 - 2x - 2 into the form a(x - h)2 + k

Answers

Answer:

A

Step-by-step explanation:

Find the vertex form of the quadratic function below.

y = x^2 - 4x + 3

This quadratic equation is in the form y = a{x^2} + bx + cy=ax  

2

+bx+c. However, I need to rewrite it using some algebraic steps in order to make it look like this…

y = a(x - h)^2 + k

This is the vertex form of the quadratic function where \left( {h,k} \right)(h,k) is the vertex or the “center” of the quadratic function or the parabola.

Before I start, I realize that a = 1a=1. Therefore, I can immediately apply the “completing the square” steps.

STEP 1: Identify the coefficient of the linear term of the quadratic function. That is the number attached to the xx-term.

STEP 2: I will take that number, divide it by 22 and square it (or raise to the power 22).

STEP 3: The output in step #2 will be added and subtracted on the same side of the equation to keep it balanced.

Think About It: If I add 44 on the right side of the equation, then I am technically changing the original meaning of the equation. So to keep it unchanged, I must subtract the same value that I added on the same side of the equation.

STEP 4: Now, express the trinomial inside the parenthesis as a square of a binomial, and simplify the outside constants.

After simplifying, it is now in the vertex form y = a{\left( {x - h} \right)^2} + ky=a(x−h)  

2

+k where the vertex \left( {h,k} \right)(h,k) is \left( {2, - 1} \right)(2,−1).

Visually, the graph of this quadratic function is a parabola with a minimum at the point \left( {2, - 1} \right)(2,−1). Since the value of “aa” is positive, a = 1a=1, then the parabola opens in upward direction.

Example 2: Find the vertex form of the quadratic function below.

The approach to this problem is slightly different because the value of “aa” does not equal to 11, a \ne 1a  

​  

=1. The first step is to factor out the coefficient 22 between the terms with xx-variables only.

STEP 1: Factor out 22 only to the terms with variable xx.

STEP 2: Identify the coefficient of the xx-term or linear term.

STEP 3: Take that number, divide it by 22, and square.

STEP 4: Now, I will take the output {9 \over 4}  

4

9

​  

 and add it inside the parenthesis.

By adding {9 \over 4}  

4

9

​  

 inside the parenthesis, I am actually adding 2\left( {{9 \over 4}} \right) = {9 \over 2}2(  

4

9

​  

)=  

2

9

​  

 to the entire equation.

Why multiply by 22 to get the “true” value added to the entire equation? Remember, I factored out 22 in the beginning. So for us to find the real value added to the entire equation, we need to multiply the number added inside the parenthesis by the number that was factored out.

STEP 5: Since I added {9 \over 2}  

2

9

​  

 to the equation, then I should subtract the entire equation by {9 \over 2}  

2

9

​  

 also to compensate for it.

STEP 6: Finally, express the trinomial inside the parenthesis as the square of binomial and then simplify the outside constants. Be careful combining the fractions.

It is now in the vertex form y = a{\left( {x - h} \right)^2} + ky=a(x−h)  

2

+k where the vertex \left( {h,k} \right)(h,k) is \left( {{{ - \,3} \over 2},{{ - 11} \over 2}} \right)(  

2

−3

​  

,  

2

−11

​  

).

Example 3: Find the vertex form of the quadratic function below.

Solution:

Factor out - \,3−3 among the xx-terms.

The coefficient of the linear term inside the parenthesis is - \,1−1. Divide it by 22 and square it. Add that value inside the parenthesis. Now, figure out how to make the original equation the same. Since we added {1 \over 4}  

4

1

​  

 inside the parenthesis and we factored out - \,3−3 in the beginning, that means - \,3\left( {{1 \over 4}} \right) = {{ - \,3} \over 4}−3(  

4

1

​  

)=  

4

−3

​  

 is the value that we subtracted from the entire equation. To compensate, we must add {3 \over 4}  

4

3

​  

 outside the parenthesis.

Therefore, the vertex \left( {h,k} \right)(h,k) is \left( {{1 \over 2},{{11} \over 4}} \right)(  

2

1

​  

,  

4

11

​  

).

Example 4: Find the vertex form of the quadratic function below.

y = 5x^2 + 15x - 5  

Solution:

Factor out 55 among the xx-terms. Identify the coefficient of the linear term inside the parenthesis which is 33. Divide it by 22 and square to get {9 \over 4}  

4

9

​  

.

Add {9 \over 4}  

4

9

​  

 inside the parenthesis. Since we factored out 55 in the first step, that means 5\left( {{9 \over 4}} \right) = {{45} \over 4}5(  

4

9

​  

)=  

4

45

​  

 is the number that we need to subtract to keep the equation unchanged.

Express the trinomial as a square of binomial, and combine the constants to get the final answer.

Therefore, the vertex \left( {h,k} \right)(h,k) is {{ - \,3} \over 2},{{ - \,65} \over 4}  

2

−3

​  

,  

4

−65

​  

.

Answer:

(x - 1 )^2 - 3

Step-by-step explanation:

( x - 1 )^2 + ( -3)

x^2 - 2x + 1 - 3

x^2 - 2x - 2

I need help asap!!!​

Answers

There are 360° total in a circle, so AB is half of the circle so it’s 180°. CBA is 180° also. 180°+55°=235°, 360-235= 125° which is AC

Shaquira is baking cookies to put in packages for a fundraiser. Shaquira has made 86 8686 chocolate chip cookies and 42 4242 sugar cookies. Shaquira wants to create identical packages of cookies to sell, and she must use all of the cookies. What is the greatest number of identical packages that Shaquira can make?

Answers

Answer: 2

Step-by-step explanation:

Given: Shaquira has made 86  chocolate chip cookies and 42 sugar cookies.

Shaquira wants to create identical packages of cookies to sell, and she must use all of the cookies.

Now, the greatest number of identical packages that Shaquira can make= GCD of 86 and 42

Prime factorization of 86 and 42:

86 = 2 ×43

42 = 2 × 3 × 7

GCD of 86 and 42 = 2   [GCD = greatest common factor]

Hence, the greatest number of identical packages that Shaquira can make =2

AB =
Round your answer to the nearest hundredth.
B
?
2
25°
С
A

Answers

Answer:

? = 4.73

Step-by-step explanation:

Since this is a right triangle we can use trig functions

sin theta = opp / hyp

sin 25 = 2 / ?

? sin 25 = 2

? = 2 / sin 25

? =4.732403166

To the nearest hundredth

? = 4.73

How do u simplify each expression by combining like terms?

Answers

Answer:

1. 8y - 9y = -1y

( 8 - 9 = -1)

3. 8a - 6 +a - 1

( i have showed the like terms here)

8a - 1a= 7a

-6 - 1 = -7

7a - 7

5. -x - 2 + 15x

( i have showed the like terms here)

-x + 15x = 14x

(x = 1)

14x + 2

7.  8d - 4 - d - 2

( i have showed the like terms here)

8d - d = 7d

-4 -2 = -6

7d - 6

8. 9a + 8 - 2a - 3 - 5a

( i have showed the like terms here)

9a - 2a - 5a = 2a

8 - 3= 5

2a + 5

prove tan(theta/2)=sin theta/1+cos theta for theta in quadrant 1 by filling in the calculations and reasons. PLEASE HELP!!!!

Answers

Answer:

See explanation

Step-by-step explanation:

We have to prove the identity

[tex]tan(\frac{\Theta }{2})=\frac{sin\Theta}{1+cos\Theta }[/tex]

We will take right hand side of the identity

[tex]\frac{sin\Theta}{1+cos\Theta}=\frac{2sin(\frac{\Theta }{2})cos(\frac{\Theta }{2})}{1+[2cos^{2}(\frac{\Theta }{2})-1]}[/tex]

[tex]=\frac{2sin(\frac{\Theta }{2})cos(\frac{\Theta }{2})}{2cos^{2}(\frac{\Theta }{2})}=\frac{sin(\frac{\Theta }{2})}{cos(\frac{\Theta }{2})}[/tex]

[tex]=tan(\frac{\Theta }{2})[/tex] [ Tan θ will be positive since θ lies in 1st quadrant ]

The equation of line WX is 2x + y = −5. What is the equation of a line perpendicular to line WX in slope-intercept form that contains point (−1, −2)?

Answers

Answer: [tex]y=\dfrac12x-\dfrac{3}{4}[/tex]

Step-by-step explanation:

Given, The equation of line WX is 2x + y = −5.

It can be written as [tex]y=-2x-5[/tex] comparing it with slope-intercept form y=mx+c, where m is slope and c is y-intercept, we have

slope of WX = -2

Product of slopes of two perpendicular lines is -1.

So, (slope of WX) × (slope of perpendicular to WX)=-1

[tex]-2\times\text{slope of WX}=-1\\\\\Rightarrow\ \text{slope of WX}=\dfrac{1}{2}[/tex]

Equation of a line passes through (a,b) and has slope m:

[tex]y-b=m(x-a)[/tex]

Equation of a line perpendicular to WX contains point (−1, −2) and has slope [tex]=\dfrac12[/tex]

[tex]y-(-2)=\dfrac{1}{2}(x-(-1))\\\\\Rightarrow\ y+2=\dfrac12(x+1)\\\\\Rightarrow\ y+2=\dfrac12x+\dfrac12\\\\\Rightarrow\ y=\dfrac12x+\dfrac12-2\\\\\Rightarrow\ y=\dfrac12x-\dfrac{3}{4}[/tex]

Equation of a line perpendicular to line WX in slope-intercept form that contains point (−1, −2) [tex]:y=\dfrac12x-\dfrac{3}{4}[/tex]

how do you solve 2m-10=44+8m

Answers

Answer:

m = -9

Step-by-step explanation:

2m-10=44+8m

Subtract 2m from each side

2m-2m-10=44+8m-2m

-10 = 44+6m

Subtract 44 from each side

-10-44 = 44-44+6m

-54 = 6m

Divide by 6

-54/6 = 6m/6

-9 = m

Answer:

solve by solving the salvation for equation don't be a slave get educated from what's gave

3/4a−16=2/3a+14 PLEASE I NEED THIS QUICK and if you explain the steps that would be geat:) Thank youuuuuuu

Answers

Answer:

360

Step-by-step explanation:

3/4a - 16 = 2/3a + 14               ⇒ collect like terms 3/4a - 2/3a = 14 + 16               ⇒ bring the fractions to same denominator9/12a - 8/12a = 30                  ⇒ simplify fraction1/12a = 30                               ⇒ multiply both sides by 12a = 30*12a = 360                                   ⇒ answer

Use the discriminant to determine the number of real solutions to the equation. −8m^2+2m=0

Answers

Answer:

discriminant is b²-4ac

= 2²-4(-8)(0)

= 0

one solution

hope this helps :)

What is the value of b?

Answers

Answer:

  55°

Step-by-step explanation:

Perhaps you want the measure of angle B. (There is no "b" in the figure.)

That measure is half the measure of the intercepted arc:

  m∠B = 110°/2 = 55°

Angle B is 55°.

Two co-interior angles
formed between the
two parallel lines are in the ratio of 11.7.
Find the measures
of angles

Answers

Answer:

110° and 70°

Step-by-step explanation:

The angles are supplementary, thus sum to 180°

sum the parts of the ratio, 11 + 7 = 18

divide 180° by 18 to find the value of one part of the ratio

180° ÷ 18 = 10° ← value of 1 part of the ratio

Thus

11 parts = 11× 10° = 110°

7 parts = 7 × 10° = 70°

The angles are 110° and 70°

I need hellp please its my last chance to become a senior please someone

Answers

Answer:

d= 6

r= 6/2

r=3

V= π. r². h

V= π . 3². 14

V= π. 9 . 14

V= π 126 cm³

V= 126 π cm³ (π not in number)

hope it helps^°^

Answer:if you use the formula it is 126 pi cm cubed

The answer is c

Step-by-step explanation:

State whether the given measurements determine zero, one, or two triangles. A = 58°, a = 25, b = 28

Answers

Answer:

1

Step-by-step explanation:

I believe it is 1. Just picture or draw a diagram of the constraints. Don't quote me on this though...

Answer:

Step-by-step explanation:

apply sine formula

[tex]\frac{a}{sin ~A} =\frac{b}{sin~B} \\\frac{25}{sin~58} =\frac{28}{sin ~B} \\sin~B=\frac{28}{25} \times sin~58\\B=sin^{-1} (\frac{28}{25} \times sin ~58)=71.77 \approx 72 ^\circ[/tex]

so third angle=180-(58+72)=180-130=50°

∠C=50°

[tex]cos ~C=\frac{a^2+b^2-c^2}{2ab} \\or ~2abcos~C=a^2+b^2-c^2\\2*25*28*cos ~50=25^2+28^2-c^2\\c^2=625+784-1400 *cos~50\\c^2=1409-899.90\\c^2=509.1\\c=\sqrt{509.1} \approx 22.56 \approx 22.6[/tex]

so one triangle is formed.

PLEASE help me with this question! No nonsense answers please. This is really urgent.

Answers

Answer:

last option

Step-by-step explanation:

Let's call the original angle x° and the radius of the circle y. The area of the original sector would be x / 360 * πy². The new angle, which is a 40% increase from x, can be represented as 1.4x so the area of the new sector is 1.4x / 360 * πy². Now, to find the corresponding change, we can calculate 1.4x / 360 * πy² ÷  x / 360 * πy² = (1.4x / 360 * πy²) * (360 * πy² / x). 360 * πy² cancels out so we're left with 1.4x / x which becomes 1.4, signifying that the area of the sector increases by 40%.

Which of the following best describes the graph shown below?
16
A1
1
14
O A This is the graph of a linear function
B. This is the graph of a one-to-one function
C. This is the graph of a function, but it is not one to one
D. This is not the graph of a function

Answers

Answer: C. It is a function, but is it not one-to-one

The vertical line test helps us see that we have a function. Note how it is not possible to draw a single straight line through more than one point on the curve. Any x input leads to exactly one y output. This graph passes the vertical line test. Therefore it is a function.

The function is not one-to-one because the graph fails the horizontal line test. Here it is possible to draw a single straight horizontal line through more than one point on the curve. The horizontal line through y = 2 is one example of many where the graph fails the horizontal line test, meaning the function is not one-to-one.

The term "one-to-one" means that each y value only pairs up with one x value. Here we have something like y = 2 pair up with multiple x values at the same time. This concept is useful when it comes to determining inverse functions.

The graph below shows Roy's distance from his office (y), in miles, after a certain amount of time (x), in minutes: Graph titled Roys Distance Vs Time shows 0 to 10 on x and y axes at increments of 1.The label on x axis is time in minutes and that on y axis is Distance from Office in miles. Lines are joined at the ordered pairs 0, 0 and 1, 1 and 2, 2 and 3, 3 and 4, 4 and 5, 4 and 6, 4 and 7, 4.5 and 7.5, 5 and 8, 6. Four students described Roy's motion, as shown in the table below: Student Description Peter He drives a car at a constant speed for 4 minutes, then stops at a crossing for 6 minutes, and finally drives at a variable speed for the next 2 minutes. Shane He drives a car at a constant speed for 4 minutes, then stops at a crossing for 2 minutes, and finally drives at a variable speed for the next 8 minutes. Jamie He drives a car at a constant speed for 4 minutes, then stops at a crossing for 6 minutes, and finally drives at a variable speed for the next 8 minutes. Felix He drives a car at a constant speed for 4 minutes, then stops at a crossing for 2 minutes, and finally drives at a variable speed for the next 2 minutes. Which student most accurately described Roy's motion? Peter Shane Jamie Felix

Answers

Answer:

Felix

Step-by-step explanation:

The graph contains 3 segments,

first one is for the first 4minutes,

second one is for the next 2 minutes (standing still)

third one is for the last 2 minutes.

Only Felix has it right, the other students use absolute time in their statements, in stead of the difference between start and end. (e.g., from 4 to 6 is 2 minutes).

The student that most accurately described Roy's motion is Felix.

How to find the function which was used to make graph?

There are many tools we can use to find the information of the relation which was used to form the graph.

A graph contains data of which input maps to which output.

Analysis of this leads to the relations which were used to make it.

We need to find the student that most accurately described Roy's motion.

Here we can see that the graph contains 3 segments, first one is for the first 4 minutes, Second one is for the next 2 minutes (standing still) and the third one is for the last 2 minutes.

Now, Only Felix has it right, the other students use absolute time in their statements, in stead of the difference between start and end.

Therefore, the student that most accurately described Roy's motion is Felix.

Learn more about finding the graphed function here:

https://brainly.com/question/27330212

#SPJ5

Given the following three points, find by hand the quadratic function they represent.
(-1,-8), (0, -1),(1,2)
(1 point)
Of(x) = -51% + 87 - 1
O f(x) = -3.2? + 4.1 - 1
Of(t) = -202 + 5x - 1
Of(1) = -3.1? + 10.1 - 1​

Answers

Answer:

The correct option is;

f(x) = -2·x² + 5·x - 1

Step-by-step explanation:

Given the points

(-1, -8), (0, -1), (1, 2), we have;

The general quadratic function;

f(x) = a·x² + b·x + c

From the given points, when x = -1, y = -8, which gives

-8 = a·(-1)² + b·(-1) + c = a - b + c

-8 =  a - b + c.....................................(1)

When x = 0, y = -1, which gives;

-1 = a·0² + b·0 + c = c

c = -1.....................................................(2)

When x = 1, y = 2, which gives;

2 = a·1² + b·1 + c = a + b + c...............(3)

Adding equation (1) to (3), gives;

-8 + 2 = a - b + c + a + b + c

-6 = 2·a + 2·c

From equation (2), c = -1, therefore;

-6 = 2·a + 2×(-1)

-2·a  = 2×(-1)+6 = -2 + 6 = 4

-2·a = 4

a = 4/-2 = -2

a = -2

From equation (1), we have;

-8 =  a - b + c = -2 - b - 1 = -3 - b

-8 + 3 = -b

-5 = -b

b = 5

The equation is therefore;

f(x) = -2·x² + 5·x - 1

The correct option is f(x) = -2·x² + 5·x - 1.

What the relation of 1/c=1/c1+1/c2 hence find c​

Answers

[tex]\frac 1c=\frac1{c_1}+\frac1{c_2} [/tex]

$\frac1c=\frac{c_1+c_2}{c_1c_2}$

$\implies c=\frac{c_1c_2}{c_1+c_2}$

SAVINGS ACCOUNT Demetrius deposits $120 into his account. One week later, he withdraws $36. Write an addition expression to represent this situation. How much higher or lower is the amount in his account after these two transactions?

Answers

Answer:

+$120 - $36

Higher by $84

Step-by-step explanation:

Addition expression is an equation without the equals to sign

$120 - $36

When the first expression was made, the account was higher by $120

After the second transaction, the account would be higher by $120 - $36 = $84

I need help fast please

Answers

Answer:

Difference : 4th option

Step-by-step explanation:

The first thing we want to do here is to factor the expression x² + 3x + 2. This will help us if it is similar to the factored expression " ( x + 2 )( x + 1 ). " The denominators will be the same, and hence we can combine the fractions.

x² + 3x + 2 - Break the expression into groups,

( x² + x ) + ( 2x + 2 ) - Factor x from x² + x and 2 from 2x + 2,

x( x + 1 ) + 2( x + 2 ) - Group,

( x + 2 )( x + 1 )

This is the same as the denominator of the other fraction, and therefore we can combine the fractions.

x - 1 / ( x + 2 )( x + 1 )

As you can see this is not any of the options present, as we have not expanded ( x + 2 )( x + 1 ). Remember previously that ( x + 2 )( x + 1 ) = x² + 3x + 2. Hence our solution is x - 1 / x² + 3x + 2, or option d.

Will give Brainliest, Please show work.

Answers

Answer:

Hi, there!!

Hope you mean the answers in the solution.

Hope it helps...

Answer:

Step-by-step explanation:

7)

JKLM is a isosceles trapezium.

KL // JM

∠K + ∠J =  180   {Co interior angles}

50 +∠J = 180

      ∠J = 180 - 50

      ∠J = 130

As it is isosceles, non parallel sides KJ = LM &  

∠L = ∠K

∠L = 50

∠M = ∠J

∠M = 130

8)JKLM is a isosceles trapezium.

KL // JM

∠K + ∠J =  180   {Co interior angles}

100 +∠J = 180

      ∠J = 180 - 100

      ∠J = 80

As it is isosceles, non parallel sides KJ = LM &  

∠L = ∠K

∠L = 100

∠M = ∠J

∠M = 80

PLEASE HELP ME !!!!!!!!!!!!!!!!!

Answers

Answer:

  2

Step-by-step explanation:

The series is a geometric series with a common ratio (r) of 1/2 and a first term (a1) of 1/2^0 = 1. The sum of such a series is given by ...

  S = a1/(1 -r) 1/(1 -1/2) = 2

The sum of the series is 2.

Other Questions
Assume that your parents wanted to have a 170,000 saved for college by your 18th birthday and they started saving on your first birthday. They saved the same amount each year on your birthday and earned 6.5% per year on their investmenets.Required:a. How much would they have to save each year to reach their goal?b. If they think you will take five years instead of four to graduate to graduate and decide to have $140,000 saved just in case, how much more would they have to save each year to reach their new goal? divide 1725 by 102 show work please Your friend wears earbuds while driving to school each day. Is this safe? Is it legal? Explain why or why not. Y * 3 = 81 please i need it for today A pair of adjacent coils has a mutual inductance of 1.5 H. If the current in one coil changes from 0 to 20 A in 0.5 s, what is the change of flux linkage with the other coil? In 1964, 12 people were killed during a tsunami at Crescent City, California. All of these fatalities were caused by the ________ wave, which was the highest in the series. The power lines are at a high potential relative to the ground, so there is an electric field between the power lines and the ground. To maximize the potential difference between one end of the fluorescent tube and the other, how should the tube be held?a. The tube should be held horizontally, parallel to the ground b. The potential difference between the ends of the tube does not depend on the tube's orientation. c. The tube should be held vertically perpendicular to the ground How the electoral college controversy during the Hayes-Tilden presidential election of 1876 was resolved? Find the distance between points K(1, 3) and L(0, 0). Round to the nearest tenth. The school's square parking lot has anarea of 3025 ft2. What is the length of eachside of the parking lot? #3: Directions: Read the paragraph and answer the question. (1) Female astronauts, lawyers, and legislators are examples of the increasing number of women in leadership roles today. (2) Margaret Thatcher, Maya Angelou, Janet Reno, and Rita Moreno achieved great success in their fields. (3) In developing nations, women are particularly needed because much work must be done. (4) The contributions of women are essential in the process of finding solutions to the worlds problems. Select the correct answer. Which numbered sentence is the topic sentence? A. 1 B. 2 C. 3 D. 4 GIVING BRAINLIEST TO THE FIRST PERSON TO ANSWER! Three stores have the same tablet computer on sale. The regular price of the tablet is $150. Store A is offering the tablet on sale at 15% off the regular price. Store B is offering a $25 coupon to be deducted from the regular price. Store C is offering a rebate of $20.00 to purchasers. Store D has the tablet on sale for $120.00. Which store is offering the tablet at the lowest cost? A. Store AB. Store BC. Store C D. Store DShow ALL work please! musah stands at the center of a rectangular field . He first takes 50 steps north, then 25 step west and finally 50 steps on a bearing of 315. How far west and how far north is Musah final point from the center? Read the excerpt from "The Royal House of Thebes."Antigone and Ismene heard with horror what Creon haddecided. To Ismene, shocking as it was, overwhelmingher with anguish for the pitiful dead body and thelonely, homeless soul, it seemed, nevertheless, thatnothing could be done except to acquiesce [to acceptwithout protest]. She and Antigone were utterly alone.All Thebes was exulting that the man who had broughtwar upon them should be thus terribly punished. "Weare women," she told her sister. "We must obey. Wehave no strength to defy the State." "Choose your ownpart," Antigone said. "I go to bury the brother I love.""You are not strong enough," Ismene cried. "Why, thenwhen my strength fails, " Antigone answered, "I will giveup." She left her sister, Ismene dared not follow her.How does Antigone's action as a rebel reveal theuniversal theme?It shows that she is tired of being alone, revealingthat loneliness is difficult for people to handle.It shows that she can break the rules for herprinciples, revealing that women are as capable asmen.It shows that she does not side with her sister,revealing that families should stick together.It shows that she pities Ismene, revealing thatunderstanding leads to right action. The government wants to set the socially optimal level of nitrogen runoff, and government regulators believe that the actual marginal benefit of pollution (MBP) is given by the estimated MBP curve. The deadweight loss associated with a quota is _____, w A consumer plays the role of:A)a wage earner.B)a saver.C)a borrower.D)All of these choices are correct. find the total area of the prism What is the energy of a photon of electromagnetic radiation with a wavelength of 963.5 nm? (c = 3.00 108 m/s, h = 6.63 1034 J s solve for x, x=2.7(35)+1/2(-9.7)*35^2 subtract c from 7, then divide b by the result